Monday, January 7, 2019

Arthimatic Aptitude Online Test For bank,RRB JE,SSC CGL More Exams



Loading...  Loading Test...

Instruction:

  • Total number of questions : 20.
  • Time alloted : 30 minutes.
  • Each question carry 1 mark, no negative marks.
  • DO NOT refresh the page.
  • All the best :-).


1.

Find the greatest number that will divide 43, 91 and 183 so as to leave the same remainder in each case.

A.
4
B.
7
C.
9
D.
13

Your Answer: Option (Not Answered)

Correct Answer: Option A

Explanation:

Required number = H.C.F. of (91 - 43), (183 - 91) and (183 - 43)

     = H.C.F. of 48, 92 and 140 = 4.


2.

Which of the following fraction is the largest ?

A.
7
8
B.
13
16
C.
31
40
D.
63
80

Your Answer: Option (Not Answered)

Correct Answer: Option A

Explanation:

L.C.M. of 8, 16, 40 and 80 = 80.

7 = 70 ;   13 = 65 ;   31 = 62
8 80 16 80 40 80

Since, 70 > 65 > 63 > 62 , so 7 > 13 > 63 > 31
80 80 80 80 8 16 80 40

So, 7 is the largest.
8


3.

.009 = .01
?

A.
.0009
B.
.09
C.
.9
D.
9

Your Answer: Option (Not Answered)

Correct Answer: Option C

Explanation:

Let .009 = .01;     Then x = .009 = .9 = .9
x .01 1

4.

The least perfect square, which is divisible by each of 21, 36 and 66 is:

A.
213444
B.
214344
C.
214434
D.
231444

Your Answer: Option (Not Answered)

Correct Answer: Option A

Explanation:

L.C.M. of 21, 36, 66 = 2772.

Now, 2772 = 2 x 2 x 3 x 3 x 7 x 11

To make it a perfect square, it must be multiplied by 7 x 11.

So, required number = 22 x 32 x 72 x 112 = 213444


5.

If x = 3 + 1 and y = 3 - 1 , then the value of (x2 + y2) is:
3 - 1 3 + 1

A.
10
B.
13
C.
14
D.
15

Your Answer: Option (Not Answered)

Correct Answer: Option C

Explanation:

x = (3 + 1) x (3 + 1) = (3 + 1)2 = 3 + 1 + 23 = 2 + 3.
(3 - 1) (3 + 1) (3 - 1) 2

y = (3 - 1) x (3 - 1) = (3 - 1)2 = 3 + 1 - 23 = 2 - 3.
(3 + 1) (3 - 1) (3 - 1) 2

x2 + y2 = (2 + 3)2 + (2 - 3)2

   = 2(4 + 3)

   = 14


Direction (for Q.No. 6):

Each of the questions given below consists of a question followed by three statements. You have to study the question and the statements and decide which of the statement(s) is/are necessary to answer the question.

6.

What is Arun's present age?

I. 

Five years ago, Arun's age was double that of his son's age at that time.

II. 

Present ages of Arun and his son are in the ratio of 11 : 6 respectively.

 III. 

Five years hence, the respective ratio of Arun's age and his son's age will become 12 : 7.

A.
Only I and II
B.
Only II and III
C.
Only I and III
D.
Any two of the three
E.
None of these

Your Answer: Option (Not Answered)

Correct Answer: Option D

Explanation:

 II. Let the present ages of Arun and his son be 11x and 6x years respectively.

  I. 5 years ago, Arun's age = 2 x His son's age.

III. 5 years hence, Arun's Age = 12
Son's age 7

Clearly, any two of the above will give Arun's present age.

Correct answer is (D).


7.

If A = x% of y and B = y% of x, then which of the following is true?

A.
A is smaller than B.
B.
A is greater than B
C.
Relationship between A and B cannot be determined.
D.
If x is smaller than y, then A is greater than B.
E.
None of these

Your Answer: Option (Not Answered)

Correct Answer: Option E

Explanation:

x% of y = x x y = y x x = y% of x
100 100

A = B.


8.

If 20% of a = b, then b% of 20 is the same as:

A.
4% of a
B.
5% of a
C.
20% of a
D.
None of these

Your Answer: Option (Not Answered)

Correct Answer: Option A

Explanation:

20% of a = b      20 a = b.
100

b% of 20 = b x 20 = 20 a x 1 x 20 = 4 a = 4% of a.
100 100 100 100


9.

3 pumps, working 8 hours a day, can empty a tank in 2 days. How many hours a day must 4 pumps work to empty the tank in 1 day?

A.
9
B.
10
C.
11
D.
12

Your Answer: Option (Not Answered)

Correct Answer: Option D

Explanation:

Let the required number of working hours per day be x.

More pumps, Less working hours per day (Indirect Proportion)

Less days, More working hours per day (Indirect Proportion)

Pumps 4 : 3 :: 8 : x
Days 1 : 2

4 x 1 x x = 3 x 2 x 8

x = (3 x 2 x 8)
(4)

x = 12.


10.

One pipe can fill a tank three times as fast as another pipe. If together the two pipes can fill the tank in 36 minutes, then the slower pipe alone will be able to fill the tank in:

A.
81 min.
B.
108 min.
C.
144 min.
D.
192 min.

Your Answer: Option (Not Answered)

Correct Answer: Option C

Explanation:

Let the slower pipe alone fill the tank in x minutes.

Then, faster pipe will fill it in x minutes.
3

1 + 3 = 1
x x 36

4 = 1
x 36

x = 144 min.


11.

A motorboat, whose speed in 15 km/hr in still water goes 30 km downstream and comes back in a total of 4 hours 30 minutes. The speed of the stream (in km/hr) is:

A.
4
B.
5
C.
6
D.
10

Your Answer: Option (Not Answered)

Correct Answer: Option B

Explanation:

Let the speed of the stream be x km/hr. Then,

Speed downstream = (15 + x) km/hr,

Speed upstream = (15 - x) km/hr.

30 + 30 = 4 1
(15 + x) (15 - x) 2

900 = 9
225 - x2 2

9x2 = 225

x2 = 25

x = 5 km/hr.


12.

A man took loan from a bank at the rate of 12% p.a. simple interest. After 3 years he had to pay Rs. 5400 interest only for the period. The principal amount borrowed by him was:

A.
Rs. 2000
B.
Rs. 10,000
C.
Rs. 15,000
D.
Rs. 20,000

Your Answer: Option (Not Answered)

Correct Answer: Option C

Explanation:

Principal = Rs. 100 x 5400 = Rs. 15000.
12 x 3


13.

A man walked diagonally across a square lot. Approximately, what was the percent saved by not walking along the edges?

A.
20
B.
24
C.
30
D.
33

Your Answer: Option (Not Answered)

Correct Answer: Option C

Explanation:

Let the side of the square(ABCD) be x metres.

Then, AB + BC = 2x metres.

AC = 2x = (1.41x) m.

Saving on 2x metres = (0.59x) m.

Saving % = 0.59x x 100 % = 30% (approx.)
2x


14.

By investing in 16% stock at 64, one earns Rs. 1500. The investment made is:

A.
Rs. 5640
B.
Rs. 5760
C.
Rs. 7500
D.
Rs. 9600

Your Answer: Option (Not Answered)

Correct Answer: Option B

Explanation:

To earn Rs. 50 , investment = Rs. 64.
3

To earn Rs. 1500, investment = Rs. 64 x 3 x 1500 = Rs. 5760.
50


15.

A man buys a watch for Rs. 1950 in cash and sells it for Rs. 2200 at a credit of 1 year. If the rate of interest is 10% per annum, the man:

A.
gains Rs. 55
B.
gains Rs. 50
C.
loses Rs. 30
D.
gains Rs. 30

Your Answer: Option (Not Answered)

Correct Answer: Option B

Explanation:

S.P. = P.W. of Rs. 2200 due 1 year hence
= Rs. 2200 x 100
100 + (10 x 1)
= Rs. 2000.

Gain = Rs. (2000 - 1950) = Rs. 50.


Direction (for Q.No. 16):
Find the odd man out.
16.

10, 25, 45, 54, 60, 75, 80

A.
10
B.
45
C.
54
D.
75

Your Answer: Option (Not Answered)

Correct Answer: Option C

Explanation:

Each of the numbers except 54 is multiple of 5.

Direction (for Q.Nos. 17 - 19):
Find out the wrong number in the given sequence of numbers.
17.

582, 605, 588, 611, 634, 617, 600

A.
634
B.
611
C.
605
D.
600

Your Answer: Option (Not Answered)

Correct Answer: Option A

Explanation:

Alternatively 23 is added and 17 is subtracted from the terms. So, 634 is wrong.

18.

36, 54, 18, 27, 9, 18.5, 4.5

A.
4.5
B.
18.5
C.
54
D.
18

Your Answer: Option (Not Answered)

Correct Answer: Option B

Explanation:

The terms are alternatively multiplied by 1.5 and divided by 3. However, 18.5 does not satisfy it.

19.

56, 72, 90, 110, 132, 150

A.
72
B.
110
C.
132
D.
150

Your Answer: Option (Not Answered)

Correct Answer: Option D

Explanation:

The numbers are 7 x 8, 8 x 9, 9 x 10, 10 x 11, 11 x 12, 12 x 13.

So, 150 is wrong.


Direction (for Q.No. 20):
Insert the missing number.
20.

8, 24, 12, 36, 18, 54, (....)

A.
27
B.
108
C.
68
D.
72

Your Answer: Option (Not Answered)

Correct Answer: Option A

Explanation:

Numbers are alternatively multiplied by 3 and divided by 2.

So, the next number = 54 ? 2 = 27.


Submit your test now to view the Results and Statistics with answer explanation.